Proving that the given set has at most $2^{n-1}$ elements












1














Let $n$ be a natural number and $X$ = {$1,2,...,n$}. For subsets $A$ and $B$ of $X$ we define $ADelta B$ to be the set of all those elements of $X$ which belong to exactly one of $A$ and $B$. Let $F$ be a collection of subsets of $X$ such that for any two distinct elements $A$ and $B$ in $F$ the set $A∆B$ has at least two elements. Show that $F$ has at most $2^{n-1}$ elements. Find all such collections $F$ with $2^{n−1}$ elements.



I have no clue on how and where to start. Please help.










share|cite|improve this question









New contributor




Anu Radha is a new contributor to this site. Take care in asking for clarification, commenting, and answering.
Check out our Code of Conduct.




















  • Hint: If $|X|=n$, then cardinality of the power set of $X$ is $2^{n}$.
    – Ashish K
    2 days ago












  • @bof Oops, my bad. You're right.
    – metamorphy
    2 days ago












  • It's not necessary, but I find it more pleasant to think of such a problem in terms of binary words (bitstrings) of length $n$ instead of subsets. You want a set of binary codewords of length $n$ such that any two different codewords differ in at least $2$ places, a so-called single error detecting code.
    – bof
    2 days ago






  • 1




    To see that you can't have more than $2^{n-1}$, observe that the $2^n$ subsets (or words) can be partitioned into pairs, so that each pair differs only in one place; thus your collection can't include more than one from each pair, so it can't contain more than half of the $2^n$ sets.
    – bof
    2 days ago










  • Hey, what happened to the answer?
    – Anu Radha
    2 days ago
















1














Let $n$ be a natural number and $X$ = {$1,2,...,n$}. For subsets $A$ and $B$ of $X$ we define $ADelta B$ to be the set of all those elements of $X$ which belong to exactly one of $A$ and $B$. Let $F$ be a collection of subsets of $X$ such that for any two distinct elements $A$ and $B$ in $F$ the set $A∆B$ has at least two elements. Show that $F$ has at most $2^{n-1}$ elements. Find all such collections $F$ with $2^{n−1}$ elements.



I have no clue on how and where to start. Please help.










share|cite|improve this question









New contributor




Anu Radha is a new contributor to this site. Take care in asking for clarification, commenting, and answering.
Check out our Code of Conduct.




















  • Hint: If $|X|=n$, then cardinality of the power set of $X$ is $2^{n}$.
    – Ashish K
    2 days ago












  • @bof Oops, my bad. You're right.
    – metamorphy
    2 days ago












  • It's not necessary, but I find it more pleasant to think of such a problem in terms of binary words (bitstrings) of length $n$ instead of subsets. You want a set of binary codewords of length $n$ such that any two different codewords differ in at least $2$ places, a so-called single error detecting code.
    – bof
    2 days ago






  • 1




    To see that you can't have more than $2^{n-1}$, observe that the $2^n$ subsets (or words) can be partitioned into pairs, so that each pair differs only in one place; thus your collection can't include more than one from each pair, so it can't contain more than half of the $2^n$ sets.
    – bof
    2 days ago










  • Hey, what happened to the answer?
    – Anu Radha
    2 days ago














1












1








1







Let $n$ be a natural number and $X$ = {$1,2,...,n$}. For subsets $A$ and $B$ of $X$ we define $ADelta B$ to be the set of all those elements of $X$ which belong to exactly one of $A$ and $B$. Let $F$ be a collection of subsets of $X$ such that for any two distinct elements $A$ and $B$ in $F$ the set $A∆B$ has at least two elements. Show that $F$ has at most $2^{n-1}$ elements. Find all such collections $F$ with $2^{n−1}$ elements.



I have no clue on how and where to start. Please help.










share|cite|improve this question









New contributor




Anu Radha is a new contributor to this site. Take care in asking for clarification, commenting, and answering.
Check out our Code of Conduct.











Let $n$ be a natural number and $X$ = {$1,2,...,n$}. For subsets $A$ and $B$ of $X$ we define $ADelta B$ to be the set of all those elements of $X$ which belong to exactly one of $A$ and $B$. Let $F$ be a collection of subsets of $X$ such that for any two distinct elements $A$ and $B$ in $F$ the set $A∆B$ has at least two elements. Show that $F$ has at most $2^{n-1}$ elements. Find all such collections $F$ with $2^{n−1}$ elements.



I have no clue on how and where to start. Please help.







combinatorics






share|cite|improve this question









New contributor




Anu Radha is a new contributor to this site. Take care in asking for clarification, commenting, and answering.
Check out our Code of Conduct.











share|cite|improve this question









New contributor




Anu Radha is a new contributor to this site. Take care in asking for clarification, commenting, and answering.
Check out our Code of Conduct.









share|cite|improve this question




share|cite|improve this question








edited 2 days ago









bof

50.5k457119




50.5k457119






New contributor




Anu Radha is a new contributor to this site. Take care in asking for clarification, commenting, and answering.
Check out our Code of Conduct.









asked 2 days ago









Anu Radha

608




608




New contributor




Anu Radha is a new contributor to this site. Take care in asking for clarification, commenting, and answering.
Check out our Code of Conduct.





New contributor





Anu Radha is a new contributor to this site. Take care in asking for clarification, commenting, and answering.
Check out our Code of Conduct.






Anu Radha is a new contributor to this site. Take care in asking for clarification, commenting, and answering.
Check out our Code of Conduct.












  • Hint: If $|X|=n$, then cardinality of the power set of $X$ is $2^{n}$.
    – Ashish K
    2 days ago












  • @bof Oops, my bad. You're right.
    – metamorphy
    2 days ago












  • It's not necessary, but I find it more pleasant to think of such a problem in terms of binary words (bitstrings) of length $n$ instead of subsets. You want a set of binary codewords of length $n$ such that any two different codewords differ in at least $2$ places, a so-called single error detecting code.
    – bof
    2 days ago






  • 1




    To see that you can't have more than $2^{n-1}$, observe that the $2^n$ subsets (or words) can be partitioned into pairs, so that each pair differs only in one place; thus your collection can't include more than one from each pair, so it can't contain more than half of the $2^n$ sets.
    – bof
    2 days ago










  • Hey, what happened to the answer?
    – Anu Radha
    2 days ago


















  • Hint: If $|X|=n$, then cardinality of the power set of $X$ is $2^{n}$.
    – Ashish K
    2 days ago












  • @bof Oops, my bad. You're right.
    – metamorphy
    2 days ago












  • It's not necessary, but I find it more pleasant to think of such a problem in terms of binary words (bitstrings) of length $n$ instead of subsets. You want a set of binary codewords of length $n$ such that any two different codewords differ in at least $2$ places, a so-called single error detecting code.
    – bof
    2 days ago






  • 1




    To see that you can't have more than $2^{n-1}$, observe that the $2^n$ subsets (or words) can be partitioned into pairs, so that each pair differs only in one place; thus your collection can't include more than one from each pair, so it can't contain more than half of the $2^n$ sets.
    – bof
    2 days ago










  • Hey, what happened to the answer?
    – Anu Radha
    2 days ago
















Hint: If $|X|=n$, then cardinality of the power set of $X$ is $2^{n}$.
– Ashish K
2 days ago






Hint: If $|X|=n$, then cardinality of the power set of $X$ is $2^{n}$.
– Ashish K
2 days ago














@bof Oops, my bad. You're right.
– metamorphy
2 days ago






@bof Oops, my bad. You're right.
– metamorphy
2 days ago














It's not necessary, but I find it more pleasant to think of such a problem in terms of binary words (bitstrings) of length $n$ instead of subsets. You want a set of binary codewords of length $n$ such that any two different codewords differ in at least $2$ places, a so-called single error detecting code.
– bof
2 days ago




It's not necessary, but I find it more pleasant to think of such a problem in terms of binary words (bitstrings) of length $n$ instead of subsets. You want a set of binary codewords of length $n$ such that any two different codewords differ in at least $2$ places, a so-called single error detecting code.
– bof
2 days ago




1




1




To see that you can't have more than $2^{n-1}$, observe that the $2^n$ subsets (or words) can be partitioned into pairs, so that each pair differs only in one place; thus your collection can't include more than one from each pair, so it can't contain more than half of the $2^n$ sets.
– bof
2 days ago




To see that you can't have more than $2^{n-1}$, observe that the $2^n$ subsets (or words) can be partitioned into pairs, so that each pair differs only in one place; thus your collection can't include more than one from each pair, so it can't contain more than half of the $2^n$ sets.
– bof
2 days ago












Hey, what happened to the answer?
– Anu Radha
2 days ago




Hey, what happened to the answer?
– Anu Radha
2 days ago










3 Answers
3






active

oldest

votes


















0














Let us write $X={1,ldots, n}$ and let $P_{n-1}$ be the collection of subsets of $X setminus {n}$. For each $S in P_{n-1}$, let us define $f(S) = S cup {n}$. Now let $P_n$ be the collection of subsets of $X$. Note that




  1. $f(S)$ is defined for each $S in P_{n-1}$;


  2. $f(S) not in P_{n-1}$ for each $S in P_{n-1}$;


  3. If $S$ and $S'$ are distinct sets in $P_{n-1}$ then $f(S) not = f(S')$;


  4. Therefore $P_{n-1}$ and ${f(S); S in P_{n-1}}$ are disjoint and of equal cardinality, and furthermore, $P_{n-1}$ and ${f(S); S in P_{n-1}}$ partition $P_n$.



So write $P_n = {S_1,S_2,ldots, S_{2^n}}$ where the $S_i$s are distinct and where $f(S_i) = S_{i+1}$ for each odd $i$; this is possible by 1. and 4. together. Then $F$ can contain at most one of $S_i$ and $S_{i+1}$ for each odd $i$, as the disjoint union of $S$ and $f(S)$ is precisely ${n}$ which has only 1 < 2 elements.






share|cite|improve this answer























  • Well, alright, the solution is fine, but does this explain the second part of the question $?$ If so, can you tell how?
    – Anu Radha
    19 hours ago












  • See below......
    – Mike
    19 hours ago



















1














I'll expand on one of bof's comments. If $n>0$ fix some $ain X$ and pair the subsets of $X$ as $y,,ycup{a}$ with $anotin y$. Not both of these are in $F$ because $yDelta(ycup{a})={a}$, so $|F|$ is at most half the number of subsets of $X$, i.e. $2^{n-1}$ as desired. (I'll leave you to consider the case $n=0$ separately.)






share|cite|improve this answer





















  • This solution is fine, too. But the second part?
    – Anu Radha
    19 hours ago



















0














As far as a collection $F$ with $2^{n-1}$ elements, let $F$ be the set of subsets of $X$ of odd cardinality. [Make sure you see why this works]



The set of subsets of $X$ of even cardinality would work too. [Make sure you see why this works] Thus the set of subsets of even cardinality has, by the above answers, only $2^{n-1}$ elements, which implies that the set $F$ of subsets of $X$ of odd cardinality has $2^{n-1}$ elements. And vice versa.






share|cite|improve this answer























  • Alright, thank you.
    – Anu Radha
    17 hours ago











Your Answer





StackExchange.ifUsing("editor", function () {
return StackExchange.using("mathjaxEditing", function () {
StackExchange.MarkdownEditor.creationCallbacks.add(function (editor, postfix) {
StackExchange.mathjaxEditing.prepareWmdForMathJax(editor, postfix, [["$", "$"], ["\\(","\\)"]]);
});
});
}, "mathjax-editing");

StackExchange.ready(function() {
var channelOptions = {
tags: "".split(" "),
id: "69"
};
initTagRenderer("".split(" "), "".split(" "), channelOptions);

StackExchange.using("externalEditor", function() {
// Have to fire editor after snippets, if snippets enabled
if (StackExchange.settings.snippets.snippetsEnabled) {
StackExchange.using("snippets", function() {
createEditor();
});
}
else {
createEditor();
}
});

function createEditor() {
StackExchange.prepareEditor({
heartbeatType: 'answer',
autoActivateHeartbeat: false,
convertImagesToLinks: true,
noModals: true,
showLowRepImageUploadWarning: true,
reputationToPostImages: 10,
bindNavPrevention: true,
postfix: "",
imageUploader: {
brandingHtml: "Powered by u003ca class="icon-imgur-white" href="https://imgur.com/"u003eu003c/au003e",
contentPolicyHtml: "User contributions licensed under u003ca href="https://creativecommons.org/licenses/by-sa/3.0/"u003ecc by-sa 3.0 with attribution requiredu003c/au003e u003ca href="https://stackoverflow.com/legal/content-policy"u003e(content policy)u003c/au003e",
allowUrls: true
},
noCode: true, onDemand: true,
discardSelector: ".discard-answer"
,immediatelyShowMarkdownHelp:true
});


}
});






Anu Radha is a new contributor. Be nice, and check out our Code of Conduct.










draft saved

draft discarded


















StackExchange.ready(
function () {
StackExchange.openid.initPostLogin('.new-post-login', 'https%3a%2f%2fmath.stackexchange.com%2fquestions%2f3062577%2fproving-that-the-given-set-has-at-most-2n-1-elements%23new-answer', 'question_page');
}
);

Post as a guest















Required, but never shown

























3 Answers
3






active

oldest

votes








3 Answers
3






active

oldest

votes









active

oldest

votes






active

oldest

votes









0














Let us write $X={1,ldots, n}$ and let $P_{n-1}$ be the collection of subsets of $X setminus {n}$. For each $S in P_{n-1}$, let us define $f(S) = S cup {n}$. Now let $P_n$ be the collection of subsets of $X$. Note that




  1. $f(S)$ is defined for each $S in P_{n-1}$;


  2. $f(S) not in P_{n-1}$ for each $S in P_{n-1}$;


  3. If $S$ and $S'$ are distinct sets in $P_{n-1}$ then $f(S) not = f(S')$;


  4. Therefore $P_{n-1}$ and ${f(S); S in P_{n-1}}$ are disjoint and of equal cardinality, and furthermore, $P_{n-1}$ and ${f(S); S in P_{n-1}}$ partition $P_n$.



So write $P_n = {S_1,S_2,ldots, S_{2^n}}$ where the $S_i$s are distinct and where $f(S_i) = S_{i+1}$ for each odd $i$; this is possible by 1. and 4. together. Then $F$ can contain at most one of $S_i$ and $S_{i+1}$ for each odd $i$, as the disjoint union of $S$ and $f(S)$ is precisely ${n}$ which has only 1 < 2 elements.






share|cite|improve this answer























  • Well, alright, the solution is fine, but does this explain the second part of the question $?$ If so, can you tell how?
    – Anu Radha
    19 hours ago












  • See below......
    – Mike
    19 hours ago
















0














Let us write $X={1,ldots, n}$ and let $P_{n-1}$ be the collection of subsets of $X setminus {n}$. For each $S in P_{n-1}$, let us define $f(S) = S cup {n}$. Now let $P_n$ be the collection of subsets of $X$. Note that




  1. $f(S)$ is defined for each $S in P_{n-1}$;


  2. $f(S) not in P_{n-1}$ for each $S in P_{n-1}$;


  3. If $S$ and $S'$ are distinct sets in $P_{n-1}$ then $f(S) not = f(S')$;


  4. Therefore $P_{n-1}$ and ${f(S); S in P_{n-1}}$ are disjoint and of equal cardinality, and furthermore, $P_{n-1}$ and ${f(S); S in P_{n-1}}$ partition $P_n$.



So write $P_n = {S_1,S_2,ldots, S_{2^n}}$ where the $S_i$s are distinct and where $f(S_i) = S_{i+1}$ for each odd $i$; this is possible by 1. and 4. together. Then $F$ can contain at most one of $S_i$ and $S_{i+1}$ for each odd $i$, as the disjoint union of $S$ and $f(S)$ is precisely ${n}$ which has only 1 < 2 elements.






share|cite|improve this answer























  • Well, alright, the solution is fine, but does this explain the second part of the question $?$ If so, can you tell how?
    – Anu Radha
    19 hours ago












  • See below......
    – Mike
    19 hours ago














0












0








0






Let us write $X={1,ldots, n}$ and let $P_{n-1}$ be the collection of subsets of $X setminus {n}$. For each $S in P_{n-1}$, let us define $f(S) = S cup {n}$. Now let $P_n$ be the collection of subsets of $X$. Note that




  1. $f(S)$ is defined for each $S in P_{n-1}$;


  2. $f(S) not in P_{n-1}$ for each $S in P_{n-1}$;


  3. If $S$ and $S'$ are distinct sets in $P_{n-1}$ then $f(S) not = f(S')$;


  4. Therefore $P_{n-1}$ and ${f(S); S in P_{n-1}}$ are disjoint and of equal cardinality, and furthermore, $P_{n-1}$ and ${f(S); S in P_{n-1}}$ partition $P_n$.



So write $P_n = {S_1,S_2,ldots, S_{2^n}}$ where the $S_i$s are distinct and where $f(S_i) = S_{i+1}$ for each odd $i$; this is possible by 1. and 4. together. Then $F$ can contain at most one of $S_i$ and $S_{i+1}$ for each odd $i$, as the disjoint union of $S$ and $f(S)$ is precisely ${n}$ which has only 1 < 2 elements.






share|cite|improve this answer














Let us write $X={1,ldots, n}$ and let $P_{n-1}$ be the collection of subsets of $X setminus {n}$. For each $S in P_{n-1}$, let us define $f(S) = S cup {n}$. Now let $P_n$ be the collection of subsets of $X$. Note that




  1. $f(S)$ is defined for each $S in P_{n-1}$;


  2. $f(S) not in P_{n-1}$ for each $S in P_{n-1}$;


  3. If $S$ and $S'$ are distinct sets in $P_{n-1}$ then $f(S) not = f(S')$;


  4. Therefore $P_{n-1}$ and ${f(S); S in P_{n-1}}$ are disjoint and of equal cardinality, and furthermore, $P_{n-1}$ and ${f(S); S in P_{n-1}}$ partition $P_n$.



So write $P_n = {S_1,S_2,ldots, S_{2^n}}$ where the $S_i$s are distinct and where $f(S_i) = S_{i+1}$ for each odd $i$; this is possible by 1. and 4. together. Then $F$ can contain at most one of $S_i$ and $S_{i+1}$ for each odd $i$, as the disjoint union of $S$ and $f(S)$ is precisely ${n}$ which has only 1 < 2 elements.







share|cite|improve this answer














share|cite|improve this answer



share|cite|improve this answer








edited yesterday

























answered yesterday









Mike

3,148211




3,148211












  • Well, alright, the solution is fine, but does this explain the second part of the question $?$ If so, can you tell how?
    – Anu Radha
    19 hours ago












  • See below......
    – Mike
    19 hours ago


















  • Well, alright, the solution is fine, but does this explain the second part of the question $?$ If so, can you tell how?
    – Anu Radha
    19 hours ago












  • See below......
    – Mike
    19 hours ago
















Well, alright, the solution is fine, but does this explain the second part of the question $?$ If so, can you tell how?
– Anu Radha
19 hours ago






Well, alright, the solution is fine, but does this explain the second part of the question $?$ If so, can you tell how?
– Anu Radha
19 hours ago














See below......
– Mike
19 hours ago




See below......
– Mike
19 hours ago











1














I'll expand on one of bof's comments. If $n>0$ fix some $ain X$ and pair the subsets of $X$ as $y,,ycup{a}$ with $anotin y$. Not both of these are in $F$ because $yDelta(ycup{a})={a}$, so $|F|$ is at most half the number of subsets of $X$, i.e. $2^{n-1}$ as desired. (I'll leave you to consider the case $n=0$ separately.)






share|cite|improve this answer





















  • This solution is fine, too. But the second part?
    – Anu Radha
    19 hours ago
















1














I'll expand on one of bof's comments. If $n>0$ fix some $ain X$ and pair the subsets of $X$ as $y,,ycup{a}$ with $anotin y$. Not both of these are in $F$ because $yDelta(ycup{a})={a}$, so $|F|$ is at most half the number of subsets of $X$, i.e. $2^{n-1}$ as desired. (I'll leave you to consider the case $n=0$ separately.)






share|cite|improve this answer





















  • This solution is fine, too. But the second part?
    – Anu Radha
    19 hours ago














1












1








1






I'll expand on one of bof's comments. If $n>0$ fix some $ain X$ and pair the subsets of $X$ as $y,,ycup{a}$ with $anotin y$. Not both of these are in $F$ because $yDelta(ycup{a})={a}$, so $|F|$ is at most half the number of subsets of $X$, i.e. $2^{n-1}$ as desired. (I'll leave you to consider the case $n=0$ separately.)






share|cite|improve this answer












I'll expand on one of bof's comments. If $n>0$ fix some $ain X$ and pair the subsets of $X$ as $y,,ycup{a}$ with $anotin y$. Not both of these are in $F$ because $yDelta(ycup{a})={a}$, so $|F|$ is at most half the number of subsets of $X$, i.e. $2^{n-1}$ as desired. (I'll leave you to consider the case $n=0$ separately.)







share|cite|improve this answer












share|cite|improve this answer



share|cite|improve this answer










answered yesterday









J.G.

23.2k22137




23.2k22137












  • This solution is fine, too. But the second part?
    – Anu Radha
    19 hours ago


















  • This solution is fine, too. But the second part?
    – Anu Radha
    19 hours ago
















This solution is fine, too. But the second part?
– Anu Radha
19 hours ago




This solution is fine, too. But the second part?
– Anu Radha
19 hours ago











0














As far as a collection $F$ with $2^{n-1}$ elements, let $F$ be the set of subsets of $X$ of odd cardinality. [Make sure you see why this works]



The set of subsets of $X$ of even cardinality would work too. [Make sure you see why this works] Thus the set of subsets of even cardinality has, by the above answers, only $2^{n-1}$ elements, which implies that the set $F$ of subsets of $X$ of odd cardinality has $2^{n-1}$ elements. And vice versa.






share|cite|improve this answer























  • Alright, thank you.
    – Anu Radha
    17 hours ago
















0














As far as a collection $F$ with $2^{n-1}$ elements, let $F$ be the set of subsets of $X$ of odd cardinality. [Make sure you see why this works]



The set of subsets of $X$ of even cardinality would work too. [Make sure you see why this works] Thus the set of subsets of even cardinality has, by the above answers, only $2^{n-1}$ elements, which implies that the set $F$ of subsets of $X$ of odd cardinality has $2^{n-1}$ elements. And vice versa.






share|cite|improve this answer























  • Alright, thank you.
    – Anu Radha
    17 hours ago














0












0








0






As far as a collection $F$ with $2^{n-1}$ elements, let $F$ be the set of subsets of $X$ of odd cardinality. [Make sure you see why this works]



The set of subsets of $X$ of even cardinality would work too. [Make sure you see why this works] Thus the set of subsets of even cardinality has, by the above answers, only $2^{n-1}$ elements, which implies that the set $F$ of subsets of $X$ of odd cardinality has $2^{n-1}$ elements. And vice versa.






share|cite|improve this answer














As far as a collection $F$ with $2^{n-1}$ elements, let $F$ be the set of subsets of $X$ of odd cardinality. [Make sure you see why this works]



The set of subsets of $X$ of even cardinality would work too. [Make sure you see why this works] Thus the set of subsets of even cardinality has, by the above answers, only $2^{n-1}$ elements, which implies that the set $F$ of subsets of $X$ of odd cardinality has $2^{n-1}$ elements. And vice versa.







share|cite|improve this answer














share|cite|improve this answer



share|cite|improve this answer








edited 18 hours ago

























answered 19 hours ago









Mike

3,148211




3,148211












  • Alright, thank you.
    – Anu Radha
    17 hours ago


















  • Alright, thank you.
    – Anu Radha
    17 hours ago
















Alright, thank you.
– Anu Radha
17 hours ago




Alright, thank you.
– Anu Radha
17 hours ago










Anu Radha is a new contributor. Be nice, and check out our Code of Conduct.










draft saved

draft discarded


















Anu Radha is a new contributor. Be nice, and check out our Code of Conduct.













Anu Radha is a new contributor. Be nice, and check out our Code of Conduct.












Anu Radha is a new contributor. Be nice, and check out our Code of Conduct.
















Thanks for contributing an answer to Mathematics Stack Exchange!


  • Please be sure to answer the question. Provide details and share your research!

But avoid



  • Asking for help, clarification, or responding to other answers.

  • Making statements based on opinion; back them up with references or personal experience.


Use MathJax to format equations. MathJax reference.


To learn more, see our tips on writing great answers.





Some of your past answers have not been well-received, and you're in danger of being blocked from answering.


Please pay close attention to the following guidance:


  • Please be sure to answer the question. Provide details and share your research!

But avoid



  • Asking for help, clarification, or responding to other answers.

  • Making statements based on opinion; back them up with references or personal experience.


To learn more, see our tips on writing great answers.




draft saved


draft discarded














StackExchange.ready(
function () {
StackExchange.openid.initPostLogin('.new-post-login', 'https%3a%2f%2fmath.stackexchange.com%2fquestions%2f3062577%2fproving-that-the-given-set-has-at-most-2n-1-elements%23new-answer', 'question_page');
}
);

Post as a guest















Required, but never shown





















































Required, but never shown














Required, but never shown












Required, but never shown







Required, but never shown

































Required, but never shown














Required, but never shown












Required, but never shown







Required, but never shown







Popular posts from this blog

Mario Kart Wii

What does “Dominus providebit” mean?

Antonio Litta Visconti Arese